LSAT and Law School Admissions Forum

Get expert LSAT preparation and law school admissions advice from PowerScore Test Preparation.

 Administrator
PowerScore Staff
  • PowerScore Staff
  • Posts: 8917
  • Joined: Feb 02, 2011
|
#26503
Complete Question Explanation

Question #15: Justify. The correct answer choice is (D).

To avoid serious cases of influenza, the author suggests that we annually immunize all high-risk individuals. However, each year’s vaccination will protect only against the strain of virus deemed most likely to be prevalent that year. On that basis, the author concludes that high-risk individuals will need to be immunized for a different strain every year.

The conclusion contains a logical gap, and a suitable prephrase is in order: what if the most prevalent strain never changes? If so, then there'd be no reason to receive a different vaccine every year. To prove the conclusion, we must establish that each year's most prevalent strain is different from the strain deemed most prevalent the year before. This prephrase agrees with answer choice (D).

Note, that despite the word "assumption" in the question stem, this is a Justify question, because the correct answer choice must allow the conclusion to be properly drawn. In other words, it must be sufficient to prove the conclusion.

Answer choice (A) is incorrect, because this would only affect the number of administered vaccines each year, not whether people need to be immunized for different strains every year.

Answer choice (B) is incorrect, because the likelihood of an epidemic has no bearing on the question of whether the vaccines need to be changed each year.

Answer choice (C) is attractive, but incorrect. If no vaccine protects against more than one strain, does that prove that we need to change the strains each year? Not unless we also assume that the most prevalent strain changes! If the most prevalent strain remains the same, we can keep using the same vaccine year after year, even if that vaccine protects against only one strain.

Answer choice (D) is the correct answer choice, as it agrees with our prephrase above.

Answer choice (E) is incorrect, because the question of side effects is extraneous to the argument.
 BoomBoom
  • Posts: 28
  • Joined: Mar 01, 2016
|
#22367
Hello,

I answered C, and I was wondering how D was the correct answer and if someone could explain why?

Thanks,

Chris
 Nikki Siclunov
PowerScore Staff
  • PowerScore Staff
  • Posts: 1362
  • Joined: Aug 02, 2011
|
#22387
Chris,

Thanks for your question. Generally speaking, we need a bit more input from you before we delve into a discussion of a particular LR question. Ultimately, it won't be us who are taking the test; it's you! Our goal is to help you understand what's going on, which is why you first need to do the following:
  • 1. Describe your approach to the stimulus. Did you understand the argument, if any, from a structural standpoint? What is the conclusion, and what evidence is the author using in support of that conclusion?

    2. Did you prephrase an answer to the question in the stem? If so, what was your prephrase?

    3. What exactly made the two answer choices you have listed particularly attractive? Did you use any question type-specific test (e.g. Assumption Negation Technique) to differentiate between them?
Thanks,
 BoomBoom
  • Posts: 28
  • Joined: Mar 01, 2016
|
#22398
Hello again,

1. In this stimulus, I initially set it up as follows,

P/Fact:A new government policy has been developed to avoid
many serious cases of influenza.

P/Fact:This goal will be accomplished by the annual vaccination of high-risk individuals: everyone 65 and older as well as anyone with a chronic disease that might cause them to experience complications from the influenza virus.

P/Fact:Each year’s vaccination will protect only against the strain of the influenza virus deemed most likely to be prevalent that year.

C:So every year it will be necessary for all high-risk individuals to receive a vaccine for a different strain of the virus.

2. My initial prephrase was: Since every year there is a new prevalent strain of influenza, a new vaccine is required every year to protect from that particular strain.

3. At first glance I kept only answer C as a possible contender. This was attractive because in my head I thought it fit well before the conclusion and made the conclusion follow logically (Each year’s vaccination will protect only against the strain of the influenza virus deemed most likely to be prevalent that year, and no vaccine for the influenza virus protects against more than one strain of that virus, so therefore every year it will be necessary for all high-risk individuals to receive a vaccine for a different strain of the virus.) When I read answer D, it felt like it was literally just repeating the premise right before the conclusion so I passed over this answer.

Perhaps you could point out where I went wrong and how to properly attack this question to get from a prephase to the correct answer.

Thanks!

Chris
 ashutosh_73
  • Posts: 14
  • Joined: Aug 05, 2023
|
#104858
Hi!
I don't understand, how ''D'' is necessary for the argument. Below is my understanding:

Premise: Each year’s vaccination will protect only against the strain of the influenza virus deemed most likely to be prevalent that year,

Conclusion: every year it will be necessary for all high-risk individuals to receive a vaccine for a different strain of the virus.

So, let say X, Y, Z, X, P, Q are the strains of viruses most likely to be prevalent in 6 consecutive years. Even though, X is repeated in two years, conclusion still follows.

That way, ''D'' should be a sufficient assumption, not necessary assumption.
Please help to get more clarity.
User avatar
 Jeff Wren
PowerScore Staff
  • PowerScore Staff
  • Posts: 385
  • Joined: Oct 19, 2022
|
#104956
Hi ashutosh,

I believe I see exactly where your confusion is in this question.

As mentioned above in the complete question explanation, this is actually a Justify question (what some people refer to as a Sufficient Assumption question) not an Assumption question (what some people refer to as a Necessary Assumption question). In other words, this question is asking for the answer that proves the conclusion rather than asking for an answer that is necessary for the argument.

People often confuse Justify and Assumption questions because Justify questions often use the words "assumed" or "assumption." Here, the clue that this is a Justify question are the words "allows the conclusion to be properly drawn," which is standard Justify language.

Get the most out of your LSAT Prep Plus subscription.

Analyze and track your performance with our Testing and Analytics Package.